Answer:
Those that will be required to file a California return are; Matty,
Strieby, Kennedy and Jackson .
Explanation:
Those that will be required to file a California return are; Matty, Strieby, Kennedy and Jackson. From the question we are informed that all of the entities involved are operating unitary Business, which is the same concept followed by California. And we know that
Unitary business can be regarded as group of persons which are related and having interdependent business activities/operations. So in this case, single California return will be filled by the entities involved.
Lonnie Johnson had an illustrious career as a nuclear engineer working on the Galileo space programme and the stealth bomber - but he is best known for creating a water pistol.
Hope this helped!
Good luck :p
~Emmy <3
Answer:
Option (D) is the right answer.
Explanation:
According to the scenario, the most appropriate answer is option ( D) because Stanley smith is a customer of the bank as he has a checking account in the bank.
While the other options are wrong because of the following reasons:
- Option (A) is incorrect because the company lucky licks Inc. is the customer of the bank, not any person.
- Option (B) is incorrect because Cara is not the customer of the bank as she only uses the bank's ATM.
- Option (C) is incorrect because Herman is not the customer of the bank as he only cashes his checks in the bank.
The firm’s marginal cost of production when the firm is producing 50 units of output is 33.33
Solution:
The production function is Q = 
The initial value is 10 units. The production value is 50 units The manufacturing cycle needs work as stated below.
Q = 
Q = 
L =
The wage rate is $15 . The following is the expense of the manufacturing process.
TC = 
TC = ![( 15 * (\frac{Q}{3.162} )^{2} ) + [ P_{k * 10}]](https://tex.z-dn.net/?f=%28%2015%20%2A%20%28%5Cfrac%7BQ%7D%7B3.162%7D%20%29%5E%7B2%7D%20%29%20%2B%20%5B%20P_%7Bk%20%2A%2010%7D%5D)
The marginal production cost is really the increase in manufacturing costs as output increases by 1 point.
As listed below, the marginal cost:
TC = ![( 15 * (\frac{Q}{3.162} )^{2} ) + [ P_{k * 10}]](https://tex.z-dn.net/?f=%28%2015%20%2A%20%28%5Cfrac%7BQ%7D%7B3.162%7D%20%29%5E%7B2%7D%20%29%20%2B%20%5B%20P_%7Bk%20%2A%2010%7D%5D)
MC =
= 
MC =
= 33.33
Answer: Crater will be bound because of Borg's apparent authority.
Explanation:
Crater Corp. will be bound to the contract since Bo Borg has the apparent authority as the acting Vice President of purchasing. Even though he went over the agreed amount that was over 2 million in the contract. Since the Shady company was unaware that he had exceeded his authority the contract will stay in place. If Shady company had of known that he did not have the final say and needed approval the result of the transaction would of been different.